LSAT and Law School Admissions Forum

Get expert LSAT preparation and law school admissions advice from PowerScore Test Preparation.

 Administrator
PowerScore Staff
  • PowerScore Staff
  • Posts: 8917
  • Joined: Feb 02, 2011
|
#79258
Complete Question Explanation

The correct answer choice is (A).

Answer choice (A): This is the correct answer choice.

Answer choice (B):

Answer choice (C):

Answer choice (D):

Answer choice (E):


This explanation is still in progress. Please post any questions below!
 sicm91
  • Posts: 12
  • Joined: Mar 23, 2021
|
#86412
Hi! Can you please explain why E is wrong?
User avatar
 Ryan Twomey
PowerScore Staff
  • PowerScore Staff
  • Posts: 141
  • Joined: Mar 04, 2021
|
#86418
Hey Sicm,


E is incorrect specifically because the passage does not say anything about the need to restrict a monopoly itself. The passage seems to be okay with monopolies existing, just not the abuse of a monopoly. The abuse of a monopoly is spelled out in the 4th paragraph: excluding competition and leverage. There is a nice example in that paragraph that will help you understand the concept of leverage.

So to sum up, E is wrong because the passage does not at all discuss anything about the need to restrict monopolies, instead focusing on the need to restrict monopoly's abuse of power.

Hope this helps.

Best,
Ryan

Get the most out of your LSAT Prep Plus subscription.

Analyze and track your performance with our Testing and Analytics Package.